Weird result in complex limitLimit of integral gives incorrect outputIntegrate returns unexpected resultLimit...

When did stoichiometry begin to be taught in U.S. high schools?

What is the strongest case that can be made in favour of the UK regaining some control over fishing policy after Brexit?

What's the metal clinking sound at the end of credits in Avengers: Endgame?

Binary Numbers Magic Trick

Please, smoke with good manners

Do I have an "anti-research" personality?

Mysql fixing root password

Python "triplet" dictionary?

How to replace the "space symbol" (squat-u) in listings?

Where does the labelling of extrinsic semiconductors as "n" and "p" come from?

Was it really necessary for the Lunar Module to have 2 stages?

Help, my Death Star suffers from Kessler syndrome!

Can fracking help reduce CO2?

Examples of non trivial equivalence relations , I mean equivalence relations without the expression " same ... as" in their definition?

Why does processed meat contain preservatives, while canned fish needs not?

Unexpected email from Yorkshire Bank

Pressure to defend the relevance of one's area of mathematics

Pawn Sacrifice Justification

Can I get candy for a Pokemon I haven't caught yet?

Is it possible to measure lightning discharges as Nikola Tesla?

What is the difference between `a[bc]d` (brackets) and `a{b,c}d` (braces)?

Does a creature that is immune to a condition still make a saving throw?

A non-technological, repeating, visible object in the sky, holding its position in the sky for hours

How to creep the reader out with what seems like a normal person?



Weird result in complex limit


Limit of integral gives incorrect outputIntegrate returns unexpected resultLimit problem calculating directional derivativeWhy won't Limit evaluate, and what can be done about itLimit of an inverse functionDoes Mathematica implement Risch algorithm? If it does, in which cases?Limit problem no longer works in Mathematica 11.1.0Evaluating integral seems incorrectReal integral giving complex resultHow to apply NIntegrate three times













1












$begingroup$


I am trying to evaluate a limit:



gamma[w_] = Sqrt[-(u*e)w^2 + I*(u*s)w];
Limit[Re[gamma[x]], {x -> DirectedInfinity[1]}]


I calculated the limit by hand, and the correct answer is (I also checked numerically for some examples of ${u,e,s}$ using the software):



$qquad frac s2 sqrt{frac ue}$



But for some reason, when using Limit, I get



{DirectedInfinity[(Sign[e]^2 Sign[u]^2)^(1/4)]}


So my questions are:



What is going here?

What issues should I be aware of when using Limit?










share|improve this question









New contributor




Villa is a new contributor to this site. Take care in asking for clarification, commenting, and answering.
Check out our Code of Conduct.







$endgroup$

















    1












    $begingroup$


    I am trying to evaluate a limit:



    gamma[w_] = Sqrt[-(u*e)w^2 + I*(u*s)w];
    Limit[Re[gamma[x]], {x -> DirectedInfinity[1]}]


    I calculated the limit by hand, and the correct answer is (I also checked numerically for some examples of ${u,e,s}$ using the software):



    $qquad frac s2 sqrt{frac ue}$



    But for some reason, when using Limit, I get



    {DirectedInfinity[(Sign[e]^2 Sign[u]^2)^(1/4)]}


    So my questions are:



    What is going here?

    What issues should I be aware of when using Limit?










    share|improve this question









    New contributor




    Villa is a new contributor to this site. Take care in asking for clarification, commenting, and answering.
    Check out our Code of Conduct.







    $endgroup$















      1












      1








      1





      $begingroup$


      I am trying to evaluate a limit:



      gamma[w_] = Sqrt[-(u*e)w^2 + I*(u*s)w];
      Limit[Re[gamma[x]], {x -> DirectedInfinity[1]}]


      I calculated the limit by hand, and the correct answer is (I also checked numerically for some examples of ${u,e,s}$ using the software):



      $qquad frac s2 sqrt{frac ue}$



      But for some reason, when using Limit, I get



      {DirectedInfinity[(Sign[e]^2 Sign[u]^2)^(1/4)]}


      So my questions are:



      What is going here?

      What issues should I be aware of when using Limit?










      share|improve this question









      New contributor




      Villa is a new contributor to this site. Take care in asking for clarification, commenting, and answering.
      Check out our Code of Conduct.







      $endgroup$




      I am trying to evaluate a limit:



      gamma[w_] = Sqrt[-(u*e)w^2 + I*(u*s)w];
      Limit[Re[gamma[x]], {x -> DirectedInfinity[1]}]


      I calculated the limit by hand, and the correct answer is (I also checked numerically for some examples of ${u,e,s}$ using the software):



      $qquad frac s2 sqrt{frac ue}$



      But for some reason, when using Limit, I get



      {DirectedInfinity[(Sign[e]^2 Sign[u]^2)^(1/4)]}


      So my questions are:



      What is going here?

      What issues should I be aware of when using Limit?







      calculus-and-analysis complex






      share|improve this question









      New contributor




      Villa is a new contributor to this site. Take care in asking for clarification, commenting, and answering.
      Check out our Code of Conduct.











      share|improve this question









      New contributor




      Villa is a new contributor to this site. Take care in asking for clarification, commenting, and answering.
      Check out our Code of Conduct.









      share|improve this question




      share|improve this question








      edited 50 mins ago









      m_goldberg

      89.3k873200




      89.3k873200






      New contributor




      Villa is a new contributor to this site. Take care in asking for clarification, commenting, and answering.
      Check out our Code of Conduct.









      asked 4 hours ago









      VillaVilla

      1083




      1083




      New contributor




      Villa is a new contributor to this site. Take care in asking for clarification, commenting, and answering.
      Check out our Code of Conduct.





      New contributor





      Villa is a new contributor to this site. Take care in asking for clarification, commenting, and answering.
      Check out our Code of Conduct.






      Villa is a new contributor to this site. Take care in asking for clarification, commenting, and answering.
      Check out our Code of Conduct.






















          1 Answer
          1






          active

          oldest

          votes


















          4












          $begingroup$

          I think it's worth reporting the issue to support. If you give appropriate assumptions, then you get your expected result:



          Limit[Re[gamma[x]], {x -> Infinity}, Assumptions -> u>0 && e>0]



          (s u)/(2 Sqrt[e u])







          share|improve this answer











          $endgroup$













          • $begingroup$
            Thank you, it worked.
            $endgroup$
            – Villa
            3 hours ago










          • $begingroup$
            Using Assumptions -> u >= 0 && e > 0 gives the same form as the OP's hand calculation.
            $endgroup$
            – Bob Hanlon
            2 hours ago












          Your Answer








          StackExchange.ready(function() {
          var channelOptions = {
          tags: "".split(" "),
          id: "387"
          };
          initTagRenderer("".split(" "), "".split(" "), channelOptions);

          StackExchange.using("externalEditor", function() {
          // Have to fire editor after snippets, if snippets enabled
          if (StackExchange.settings.snippets.snippetsEnabled) {
          StackExchange.using("snippets", function() {
          createEditor();
          });
          }
          else {
          createEditor();
          }
          });

          function createEditor() {
          StackExchange.prepareEditor({
          heartbeatType: 'answer',
          autoActivateHeartbeat: false,
          convertImagesToLinks: false,
          noModals: true,
          showLowRepImageUploadWarning: true,
          reputationToPostImages: null,
          bindNavPrevention: true,
          postfix: "",
          imageUploader: {
          brandingHtml: "Powered by u003ca class="icon-imgur-white" href="https://imgur.com/"u003eu003c/au003e",
          contentPolicyHtml: "User contributions licensed under u003ca href="https://creativecommons.org/licenses/by-sa/3.0/"u003ecc by-sa 3.0 with attribution requiredu003c/au003e u003ca href="https://stackoverflow.com/legal/content-policy"u003e(content policy)u003c/au003e",
          allowUrls: true
          },
          onDemand: true,
          discardSelector: ".discard-answer"
          ,immediatelyShowMarkdownHelp:true
          });


          }
          });






          Villa is a new contributor. Be nice, and check out our Code of Conduct.










          draft saved

          draft discarded


















          StackExchange.ready(
          function () {
          StackExchange.openid.initPostLogin('.new-post-login', 'https%3a%2f%2fmathematica.stackexchange.com%2fquestions%2f197263%2fweird-result-in-complex-limit%23new-answer', 'question_page');
          }
          );

          Post as a guest















          Required, but never shown

























          1 Answer
          1






          active

          oldest

          votes








          1 Answer
          1






          active

          oldest

          votes









          active

          oldest

          votes






          active

          oldest

          votes









          4












          $begingroup$

          I think it's worth reporting the issue to support. If you give appropriate assumptions, then you get your expected result:



          Limit[Re[gamma[x]], {x -> Infinity}, Assumptions -> u>0 && e>0]



          (s u)/(2 Sqrt[e u])







          share|improve this answer











          $endgroup$













          • $begingroup$
            Thank you, it worked.
            $endgroup$
            – Villa
            3 hours ago










          • $begingroup$
            Using Assumptions -> u >= 0 && e > 0 gives the same form as the OP's hand calculation.
            $endgroup$
            – Bob Hanlon
            2 hours ago
















          4












          $begingroup$

          I think it's worth reporting the issue to support. If you give appropriate assumptions, then you get your expected result:



          Limit[Re[gamma[x]], {x -> Infinity}, Assumptions -> u>0 && e>0]



          (s u)/(2 Sqrt[e u])







          share|improve this answer











          $endgroup$













          • $begingroup$
            Thank you, it worked.
            $endgroup$
            – Villa
            3 hours ago










          • $begingroup$
            Using Assumptions -> u >= 0 && e > 0 gives the same form as the OP's hand calculation.
            $endgroup$
            – Bob Hanlon
            2 hours ago














          4












          4








          4





          $begingroup$

          I think it's worth reporting the issue to support. If you give appropriate assumptions, then you get your expected result:



          Limit[Re[gamma[x]], {x -> Infinity}, Assumptions -> u>0 && e>0]



          (s u)/(2 Sqrt[e u])







          share|improve this answer











          $endgroup$



          I think it's worth reporting the issue to support. If you give appropriate assumptions, then you get your expected result:



          Limit[Re[gamma[x]], {x -> Infinity}, Assumptions -> u>0 && e>0]



          (s u)/(2 Sqrt[e u])








          share|improve this answer














          share|improve this answer



          share|improve this answer








          edited 14 mins ago









          m_goldberg

          89.3k873200




          89.3k873200










          answered 3 hours ago









          Carl WollCarl Woll

          76.3k3100200




          76.3k3100200












          • $begingroup$
            Thank you, it worked.
            $endgroup$
            – Villa
            3 hours ago










          • $begingroup$
            Using Assumptions -> u >= 0 && e > 0 gives the same form as the OP's hand calculation.
            $endgroup$
            – Bob Hanlon
            2 hours ago


















          • $begingroup$
            Thank you, it worked.
            $endgroup$
            – Villa
            3 hours ago










          • $begingroup$
            Using Assumptions -> u >= 0 && e > 0 gives the same form as the OP's hand calculation.
            $endgroup$
            – Bob Hanlon
            2 hours ago
















          $begingroup$
          Thank you, it worked.
          $endgroup$
          – Villa
          3 hours ago




          $begingroup$
          Thank you, it worked.
          $endgroup$
          – Villa
          3 hours ago












          $begingroup$
          Using Assumptions -> u >= 0 && e > 0 gives the same form as the OP's hand calculation.
          $endgroup$
          – Bob Hanlon
          2 hours ago




          $begingroup$
          Using Assumptions -> u >= 0 && e > 0 gives the same form as the OP's hand calculation.
          $endgroup$
          – Bob Hanlon
          2 hours ago










          Villa is a new contributor. Be nice, and check out our Code of Conduct.










          draft saved

          draft discarded


















          Villa is a new contributor. Be nice, and check out our Code of Conduct.













          Villa is a new contributor. Be nice, and check out our Code of Conduct.












          Villa is a new contributor. Be nice, and check out our Code of Conduct.
















          Thanks for contributing an answer to Mathematica Stack Exchange!


          • Please be sure to answer the question. Provide details and share your research!

          But avoid



          • Asking for help, clarification, or responding to other answers.

          • Making statements based on opinion; back them up with references or personal experience.


          Use MathJax to format equations. MathJax reference.


          To learn more, see our tips on writing great answers.




          draft saved


          draft discarded














          StackExchange.ready(
          function () {
          StackExchange.openid.initPostLogin('.new-post-login', 'https%3a%2f%2fmathematica.stackexchange.com%2fquestions%2f197263%2fweird-result-in-complex-limit%23new-answer', 'question_page');
          }
          );

          Post as a guest















          Required, but never shown





















































          Required, but never shown














          Required, but never shown












          Required, but never shown







          Required, but never shown

































          Required, but never shown














          Required, but never shown












          Required, but never shown







          Required, but never shown







          Popular posts from this blog

          Gersau Kjelder | Navigasjonsmeny46°59′0″N 8°31′0″E46°59′0″N...

          What is the “three and three hundred thousand syndrome”?Who wrote the book Arena?What five creatures were...

          Are all UTXOs locked by an address spent in a transaction?UTXO all sent to change address?Signing...